Criminal Procedures Final

¡Supera tus tareas y exámenes ahora con Quizwiz!

Factors determining violation of speedy trial

length of delay, gov reason, assertion of right, prejudiced to the defendant

Need consent or warrant to search

curtilage

Argersinger v Hamilton

: A criminal defendant charged with any offense punishable by imprisonment is entitled to an attorney under the Sixth Amendment. At common law in England, the right to counsel was guaranteed in misdemeanor cases but limited in felony cases. The Sixth Amendment to the United States Constitution expanded that right, and the language and subsequent case law do not support the notion that the Sixth Amendment revoked the right to counsel for misdemeanor offenses. The fact that offenses carrying a sentence of less than six months imprisonment may be tried before a judge instead of a jury does not mean that there is no right to counsel. An attorney ensures a defendant receives a fair trial. Indeed, the importance of an attorney in a criminal trial is evidenced by the almost universal use of attorneys by governments and criminal defendants with money. Prosecutions for minor offenses may involve complicated legal and constitutional issues. Further, an attorney can make sure that a defendant's guilty plea is knowing and voluntary. Courts facing an extremely high number of misdemeanor prosecutions may be forced to favor speed and efficiency over justice and individualized attention. As a result, defendants charged with misdemeanors are five times more likely to have the charges dropped if they have an attorney. Thus, unless the defendant has made a full and voluntary waiver of the right, any defendant charged with a crime punishable by imprisonment has a right to counsel. • NOTES: Right to attorney in any criminal prosecution. Likening right to council to right to jury, but that shouldn't be the same thing. Right to attorney is a fundamental right. How crimes are classified is up to the state. Overall, states are obligated to provide council if you are facing jail time. • ADDED: this case incorporate powell decision. Right to council slows the process down. Court concerned about liberty interests. Incarceration is removal of liberty so get council. If you need extra expertise, the atate has to provide that. ENDNOTES. Footnotes for Gideon. • NOTES: FOOTNOTES about indigent defendant.

Massiah v US

: A suspect's Fifth and Sixth Amendment rights are violated where he has been indicted, he has invoked his right to counsel, and federal agents have deliberately elicited incriminating statements from him in the absence of his retained counsel. In Spano v. New York (1959), the defendant had already been indicted at the time of his confession. The Court's opinion relied on the totality of the circumstances surrounding the interrogation when it held that the confession had been improperly admitted. However, the concurring opinions all emphasized the absence of Spano's attorney when he was interrogated by the police. In addition, in Powell v. Alabama (1932), the Court held that from the time of a defendant's arraignment to the beginning of trial, a defendant is as much entitled to the aid of counsel as he is during the trial itself. Therefore, Massiah's constitutional rights were violated when his own incriminating statements, intentionally elicited by the police after he had invoked his right to counsel, were introduced against him at trial. NOTES: already indicted, so 6th amendment right to counsel. Attaches during gov attempts to elicit incriminating statements from you. Person eliciting statements is the codefendant.--> now a gov agent when he helps the police here. Undercover operations with deliberately eliciting info post-first critical stage not allowed.

Brown v Mississippi

: Confessions induced by violence are not consistent with the Due Process Clause and such evidence is therefore inadmissible at trial. Severe beatings to garner a confession clearly violate fundamental principles of justice and therefore amount to a violation of due process. Where such confessions are the only evidence against the defendants, as is the case here, the result is simply the pretense of a fair trial. The trial court had sufficient evidence that the confessions were the result of coercion and brutality and the court wrongly permitted the confessions to be introduced into evidence. • NOTES: Confessions obtained through physical coercion are not admissible. WHY? Inherently unreliable confessions. Whose responsibility to make sure it's excluded? Judge, on their own motion when apparent it was obtained through torture. Inadmissible for all purposes for all time. Inadmissible against anyone, including people who aren't the defendant whose rights have been violated.

U.S. v Chronic

: In the absence of a showing of particularized errors by defense counsel, a defendant claiming a violation of the Sixth Amendment right to the effective assistance of counsel must demonstrate that the totality of the circumstances supports a presumption of ineffective assistance. The court of appeals made no finding that Cronic's appointed counsel committed any errors or that his performance overall prejudiced Cronic's defense. The court of appeals concluded that no such findings were necessary under circumstances that generally hinder the ability of counsel to prepare a defense. The court of appeals considered the time allowed to prepare for trial, the experience of Cronic's attorney, the seriousness of the charges and the complexity of potential defenses, and the ability of Cronic's attorney to access witnesses prior to trial. The Sixth Amendment right to counsel guarantees access to effective assistance of counsel. Effective assistance requires the involvement of counsel acting as an advocate for the defendant. When the adversarial process is effectively carried out, the defendant has received the effective assistance of counsel. Only when defense counsel is so ineffective as to deprive a trial of its adversarial character has the right to counsel been violated. The Sixth Amendment is generally not implicated when a defendant has received a fair trial. We presume that defense counsel is competent and place the burden upon the defendant to demonstrate that he has been deprived of effective assistance. In some cases, circumstances will amount to a complete deprivation of counsel irrespective of the skill of a defense attorney. For example, in Powell v. Alabama, 287 U.S. 45 (1932), we found ineffective assistance of counsel when the court appealed to the local bar for representation six days prior to trial and no attorney stepped forward until an out-of-state attorney appeared on the day of trial. Under those circumstances, it was so unlikely that defense counsel could have performed effectively that the Court found the trial inherently unfair. Similar circumstances will support a presumption of ineffective assistance of counsel. In the absence of circumstances that give rise to such a presumption, a claim of ineffective assistance must be supported by evidence related to the actual performance of counsel. The question in this case is whether the factors considered by the court of appeals present circumstances that would support the presumption of ineffectiveness. The court of appeals gave substantial weight to the disparity between the government's preparation time and the defense's preparation time. As a practical matter, the time the government spent preparing the evidence of complex financial transactions actually simplified the role of defense counsel. In addition, Cronic's defense theory was relatively simple. His defense did not turn on the authenticity of the illegal transactions, but upon his intent to commit fraud. The fact that Cronic's attorney was youthful might be relevant to evaluating his performance, but that fact alone does not support a presumption of ineffective assistance. Similarly, considerations of the seriousness of the charges, the complexity of the defense, and counsel's access to witnesses may bear upon an evaluation of counsel's performance but they do not inherently establish a lack of effective representation. The factors considered by the court of appeals do not establish that Cronic's attorney failed to effectively perform as an advocate in the adversarial process. Only by illustrating particular errors can Cronic make a case for ineffective assistance in violation of his Sixth Amendment rights • NOTES: length of time lawyer practicing BCUZ gotta learn sometime. Error is not a judgment issue. Conduct has to fall below that of a reasonable attorney to be an error.

U.S. v Russell

: The provision by law enforcement officials of some thing necessary to facilitate the commission of a crime does not amount to entrapment. The affirmative defense of entrapment bars law enforcement from inducing an otherwise innocent person to commit a criminal act. The defense was first acknowledged in Sorrells v. United States, 287 U.S. 435 (1932). The majority opinion viewed the defendant's predisposition toward committing the crime as the pivotal inquiry into whether the defendant was entitled to assert the defense of entrapment. A concurring opinion argued that the critical inquiry should focus on the extent to which the conduct of law enforcement officials provoked the criminal act. In Sherman v. United States356 U.S. 369 (1958), the majority again placed a premium on the predisposition of the defendant to commit the crime charged. A concurring opinion queried whether the conduct of law enforcement officers constituted an inappropriate exercise of governmental authority. Russell asks the Court to reevaluate the established doctrine of the entrapment defense. Russell asserts that the extent of the undercover agent's involvement in the production of the methamphetamine that led to his conviction violated constitutional due process protections. Russell proposes that the goal of deterring unlawful police conduct which underlies the exclusionary rule should apply with equal force to the entrapment defense. Russell's proposal, however, goes well beyond the effect of the exclusionary rule. Rather than excluding evidence from use in support of prosecution, his rule would prohibit prosecution altogether. His argument is further flawed by the fact that the purpose of the exclusionary rule is to deter government violations of its own laws, whereas the undercover officer in Russell's case broke no law and committed no violation of Russell's constitutional rights. Russell's proposed rule would bar prosecution whenever a government agent has provided some element that the defendant could not have independently obtained and, without which, could not have committed the crime charged. Setting aside the difficulties inherent in implementing the rule Russell proposes it nonetheless appears that he would not be entitled to claim the entrapment defense in any case. The evidence demonstrates that acquiring the ingredient supplied by the undercover agent was difficult, but not beyond the realm of possibility. To the contrary, the record shows that Russell did, in fact, successfully acquire the ingredient from other sources. Fifth Amendment due process requirements are violated by conduct that so offends notions of fundamental fairness as to disturb generally accepted concepts of justice. This is not such a case. The undercover officer provided a legally obtainable chemical that has legitimate commercial uses apart from the manufacture of methamphetamine. In recognition of the difficulty of obtaining evidence of illegal drug manufacture, the undercover infiltration of manufacturing operations has become an acceptable procedure. If infiltration is acceptable, then it stands to reason that providing some thing of interest to drug manufacturers is equally permissible. In the alternative to his constitutional argument, Russell urges us to adopt the minority views expressed in the concurrences to Sherman and Sorrells and make the inquiry into entrapment focus on the conduct of law enforcement rather than the predisposition of the defendant. These cases represent longstanding precedent, so we leave it to the legislature to determine their continuing validity should it choose to take up the question. Our rulings in entrapment cases should not be read to bestow judicial authority to gut prosecutions whenever a judge feels that law enforcement has overstepped its bounds. The defense applies only to those rare situations in which the government has actively induced an otherwise law abiding citizen to engage in self-incriminating activity. The use of deception or the facilitation by law enforcement of criminal activities does not give rise to a valid claim for the entrapment defense • NOTES: entrapment. When gov participates too much in the criminal activity, that is entrapment. He brought the ingredient for him to make the meth, he was too involved. BUT not entrapment. Can't plant the idea that they're going to commit the crime. He had already gotten the propanone before and after the officer gave it to him so not entrapment. Only when gov actually implants the idea of criminal activity in the mind of the person tha the gov is guilty of entrapment. • ADDED: entrapment definition—idea of entrapment is where the gov coerces an otherwise innocent individual into provoking them to commit a crime that they would otherwise have not committed. Footnote on 2nd page. • NOTES: otherwise innocent individual is provoked by police= entrapment.

barker v wingo

: To determine whether or not a defendant has been denied his right to a speedy trial, courts must look at both the actions of the prosecution and the defendant. The courts cannot establish a timeframe defining what "speedy" means because such rule making remains the power of the legislatures. Furthermore, courts cannot hold that the right to a speedy trial can only be violated in cases where the defendant asserts that right. No precedence supports the position that there is an assumed waiver of constitutional rights. The right to a speedy trial differs from other constitutional rights because it often protects the rights of society and not the rights of the defendant, because deprivation of the right often benefits the defendant, and because the right is inherently vague. For these reasons, a balancing approach is preferable to a bright-line rule. While none are necessary or sufficient to finding a deprivation of the right to a speedy trial, courts must consider four factors in their balancing analysis: (1) the length of the delay; (2) the government's reason for the delay; (3) whether and how the defendant asserted his right to a speedy trial; and (4) whether the defendant was prejudiced by the delay. In this case, Barker's right to a speedy trial was not violated. It is true that there was an incredible delay in Barker's case with over five years elapsing between his arrest and his trial, and most of this time was attributable to the state's inability to create a case against Barker. However, Barker was not prejudiced by the delay. He spent most of the time between his arrest and his trial out on bail, and he was not injured by witnesses who forgot key pieces of information. In addition, Barker did not want a speedy trial. He took a chance that if Manning was acquitted he would never be tried and thus he did not want his trial to commence until Manning's ended. Barker had counsel throughout the many continuance requests and it was not until he realized that Manning was not going to be acquitted that he began to assert his right to a speedy trial. Therefore, in the final balancing analysis, Barker was not deprived of his due process right. NOTES: while there is a right to a speedy trial, it is incumbent upon defendant to show right has been compromised. Barker test: 4 factors to consider if 6th amendment rights have been violated: length of delay (case by case, period of time after which it has been presumptively violated), government's reason for delay (have to assign blame for the delay, significant reason), time and manner which defendant has asserted his right (can't claim right to speedy trial without explaining this, assert that a certain amount of time has passed), degree of prejudice to the defendant which the delay has caused (out of work). No prosecutions right to a speedy trial. Delay of more than 12 months is presumptively unconstitutional.

miranda v arizona

: Without certain hallmark warnings regarding the right to remain silent and the right to counsel, statements made during custodial interrogation are inadmissible at trial. Custodial interrogation occurs if law-enforcement officers question a person who has been arrested and taken into custody, or otherwise deprived of freedom of action in any significant way. Custodial interrogation is inherently coercive. Cutting suspects off from contact with the outside world creates an environment ripe for intimidation. Acting within a closed, hostile environment, police officers can prey upon individual weaknesses at the expense of individual liberties. In each of the four cases, the risk of psychological intimidation was plain. Miranda suffered from mental illness and Stewart had dropped out of school in the sixth grade. Due to these concerns, the Constitution requires a number of procedural safeguards that relieve some of the pressure put upon people in custodial interrogation. Prior to any questioning while in custody, a person must be informed of the right to remain silent, and that anything said can be used as evidence of guilt. The arrestee must also be notified of the right to an attorney, and that if the arrestee can't pay for an attorney, one will be provided for free. Any waiver of these rights must be voluntary. If a person refuses to be questioned at any point, law enforcement cannot continue the questioning. The fact that a person answers some questions doesn't mean the person waives the right to consult with an attorney or to stop the interrogation at a later point. The statements of all four men were inadmissible because they were not informed of their rights. • NOTES: Not entitled to miranda WHENEVER they encounter police. 2 elements before 5th amendment attaches= be in police custody and be under interrogation (must have both). Purpose of right to counsel= people are losing constitutional rights because of lack of info during a critical stage in the proceedings and counteract inherent coercion in the custody and interrogation environment. Rights are your constitutional rights, but does the constitution require them to be told? NO. Miranda is protective/a prophylactic device, created by SCOTUS, to protect our rights. Miranda is so entrenched in the US that it is part of the constitution. • Quarles, public safety exception imperative to find gun to protect innocents, that overcomes miranda. Dire emergency needed to apply. • SCOTUS indicates that under miranda, normal circumstances read warnings and then asks for lawyers. ALL QUESTIONING MUST STOP until the attorney is present or the defendant initiates discussion. • Routine motor vehicle stops exception officer doing DUI stop, no miranda until arrested. Pulled over for a normal thing, unduly cumbersome to use miranda.

Strickland v Washington

A defendant has suffered from the ineffective assistance of counsel when the attorney has not acted as a reasonably competent attorney, and there is a reasonable probability that absent these errors the result of the proceeding would have been different. Addressing the first element, there is no set of rules that will establish whether an attorney acted reasonably. Instead, all of the facts, as they existed at the time counsel made his decision, must be considered. There is a strong presumption that an attorney has acted reasonably. Otherwise, any defendant, upon conviction, could challenge the sound trial strategy of his attorney since it would be easy for a court to decide that an unsuccessful act or omission was unreasonable. Addressing the second element, the purpose of Sixth Amendment right to counsel is to ensure a fair trial in order to justify reliance on the outcome of the proceedings. Therefore, any error on the part of counsel must be prejudicial to the defendant to constitute the ineffective assistance of counsel. The burden is on the defendant to prove that he was harmed by his attorney's conduct and that there is a reasonable probability that absent the attorney's errors, the outcome would have been different. In this case, there was no ineffective assistance of counsel. Counsel's decisions were all based on professionally reasonable judgments. The record shows that Washington's attorney made the strategic decision to argue mitigating circumstances due to severe emotional distress. Counsel's decisions were all consistent with this case theory. Furthermore, the record does not indicate that the lawyer's sense of hopelessness distorted his professional judgment. In addition, Washington was not prejudiced by his lawyer's actions. There is no reasonable probability that the omitted evidence would have changed the outcome. The aggravating circumstances outweighed the mitigating factors to such an extent that any character testimony would not have changed the sentencing. Therefore, Washington was not denied his Sixth Amendment right to counsel and he received a fair trial • ADDS: Chronic isn't end analysis. Can't start if there's no error, but need to show if there's going to be a different outcome reasonably. NOT a but for test. Can't prove what a jury would have done if the mistake hadn't happened. Was the jury prejudiced and was there a substantial likelihood that the outcome would have been different?

Florida v Jardines

A search occurs under the Fourth Amendment when the government obtains information by physically intruding on an individual's person, house, or belongings. Although Katz v. United States, 389 U.S. 347 (1967), permits the obtaining of information outside of a traditional search in areas where an individual does not have a reasonable expectation of privacy, Katz "does not subtract anything from the [Fourth] Amendment's protections when the Government [engages] in a physical intrusion of a constitutionally protected area." For example, using a drug-sniffing dog on a homeowner's porch to investigate the contents of the home does not invoke Katz, but rather is a search within the meaning of the Fourth Amendment. The use of a trained police dog with the objective of uncovering evidence goes beyond the implied license to go up to a house and, for example, drop off mail or knock on the door. It is a physical intrusion on the homeowner's property. Accordingly, in this case, the detective's use of the drug-sniffing dog to detect marijuana constituted a warrantless search of Jardines's porch—the curtilage of his home—in violation of his Fourth Amendment rights. The detective obtained information by physically intruding on Jardines's property, so there is no need to determine whether Jardines's expectation of privacy was intruded upon under Katz. • NOTES: Unverified tip which police go and investigate 1 month later. Surveil for 15mins and see nothing. Dog alerts for drugs and then they get the warrant. So guy says that since they didn't have a warrant when the dog was sniffing. A dog sniff is a search under the 4th amendment. Need a search warrant. WHY? Curtilage. Home has highest expectation of privacy including the curtilage. Porch is protected. Curtilage intimately linked to home both physically and psychologically. Constitutionally protected area so it was a search. Dog sniffing on property exceeds the license a person would normally give on their property. • ADDED: curtilage definition. Back porch. Secondary problem with case. Gov actors come on porch everyday (postal). When gov comes on your property and deviates from a specific purpose, they are not legally there. • NOTES: scalia says curtilage is extremely well-defined (not true). There by consent or invitation. Violating the condition on which they came onto the property. Scalia says the drug-sniffing dog is a search, which a drug-sniffing dog has never been considered a search before.

Colorado v Connelly

A statement made by a mentally ill person is not involuntary for purposes of the Due Process Clause if there is no coercive behavior by police. The Due Process Clause forbids the government from "depriv[ing] any person of life, liberty, or property, without due process of law." Coercive interrogation techniques have been held to violate the Due Process Clause in Brown v. Mississippi, 297 U.S. 278 (1936), and other cases. Nevertheless, case law makes clear that there must be coercion and overreaching by police for a violation to occur. While courts have considered the mental state of a suspect in determining whether some form of psychological coercion has occurred, the suspect's mental state alone does not render a statement involuntary for purposes of the Due Process Clause. Suppressing such statements would have no deterrent effect on constitutional violations by police and would force courts to assess a suspect's subjective mental state even where there has been no police misconduct whatsoever. Thus, a confession may not be found involuntary in violation of the Due Process clause without some element of police coercion. In this case, Connelly's statements might be unreliable and therefore inadmissible under the state rules of evidence, but there was no violation of the Due Process Clause. NOTES: state's burden to prove that the waiver occurred. Standard is clear and convincing evidence. SCOTUS says: preponderance of evidence (more likely than not). Test for voluntariness—only important if there's been some gov action. Officer didn't do anything. If a gov action, then it can be involuntary.

brady v us

A waiver of an individual's constitutional rights must not only be voluntary, but must be knowing, intelligent acts done with sufficient awareness of the relevant circumstances and likely consequences. The prosecution often utilizes plea bargaining to obtain guilty pleas to reduce the burdens on the court, staff, and community. Moreover, in many cases the defendant may never plead guilty unless assured that the plea will result in a lesser penalty than he might receive after a jury trial and subsequent conviction. The state may not, however, obtain a plea by actual or threatened physical harm or by mental coercion from the defendant. But that did not occur here. Guilty pleas are not unconstitutional because, on one hand, the criminal law extends to a judge or jury a range of choices in setting the sentence in individual cases and, on the other, because the prosecution and defendant both find advantages in avoiding the possibility of the maximum penalty authorized by law. Nearly 75 percent of the criminal convictions in the United States are secured by guilty pleas entered by the defendants. It is not unconstitutional for a state to provide a benefit to the defendant which also happens to be a benefit for the state as well. In Bram v. United States, 168 U.S. 532 (1897), the Court held that the admissibility of a defendant's confession depended on whether it was compelled within the meaning of the Fifth Amendment. To be admissible, the confession must be "free and voluntary," i.e. must not be obtained by use of threats or violence or by direct or indirect promises of any sort. Even a suggested promise of leniency was deemed sufficient to bar the confession, because, at the time of the confession and subsequent implicit promise, the defendant was too sensitive to such an inducement. Here, Brady was not in such a position. Rather, Brady was represented by competent counsel and advised of the advantages and disadvantages of going to trial and of pleading guilty. Brady's plea was entered in open court and before a judge sensitive to the requirements of law with respect to guilty pleas. It was voluntary. A guilty plea is voluntary if it is entered by a defendant who is fully aware of the direct consequences, and not induced by threats or promises. Under this standard, a plea of guilty is not invalid merely because it is entered to avoid the possibility of the death penalty. Brady's plea was also intelligently made. Brady was advised by counsel and made aware of the charges against him. There is nothing to indicate that he was incompetent or otherwise not in control of his mental faculties. NOTES: the court doesn't require every guilty plea to be overturned when the fear of death was a factor. Plea voluntary, intelligent, knowingly. Can't be sentenced to death without a jury. If defendant waives right to jury trial they can cheat the system and avoid the death penalty.

kyllo v US

All details of what transpires in a home are intimate details and protected by the Fourth Amendment unless they are freely observable by the public. Using a device that is not publicly available to see details of a private home that would be undiscoverable without physically entering the home constitutes a Fourth Amendment search. This type of search is presumptively unreasonable without a warrant. Here, although there was no physical intrusion on Kyllo's home, the thermal-imaging device was used to determine what was happening in his home. The warrantless search of Kyllo's home using the thermal-imaging device was unconstitutional. Accordingly, the judgment is reversed, and the case is remanded for a determination of whether there was sufficient evidence without the thermal imaging to support the warrant. • NOTES: 4 part test whether or not tech violates the 4th amendment: 1) sense-enhancing tech 2) needs to gather info regarding the interior of a home 3) wouldn't get this info without physical intrusion in a constitutionally protected area 4) something that the public wouldn't be able to get. Need a warrant if any of this is violated. • NOTES: when tech used is itself using sense-enhancing tech that you couldn't figure it out yourself? KNOW 4 PT test. Sense-enhancing devices all over (glass, hearing aids, etc). Sense enhancing device is not readily publicly available (main important part)

Brewer v Williams

An effective waiver requires actual relinquishment of a right. If a defendant consistently relies on the advice of counsel in dealing with the police, any suggestion that he waived his right to counsel is refuted. Under the Sixth and Fourteenth Amendments, a person has a right to counsel at or after the time that judicial proceedings have been initiated against him. Furthermore, the police may not interrogate a suspect alone after he has invoked his right to counsel. In this case, judicial proceedings had been initiated against Williams at the time of his car trip back to Des Moines. The officer's "Christian burial speech" amounted to interrogation because the officer himself testified to the fact that his statements were intended to elicit information from Williams. Williams invoked his right to counsel throughout his ordeal. He contacted his attorney before turning himself in, he continued to employ the advice of counsel by remaining silent, and he even told the police he would tell them everything but only after he consulted with his attorney. Despite this, the officer elicited incriminating statements from Williams without first reading him his Miranda rights or ascertaining whether Williams wished to waive his right to counsel. Under such facts, no effective waiver took place. NOTES: arraigned, post critical stage, 6th amendment. Once attached, police can't deliberately elicit incriminating info. Officer never asked a question, told him not to respond. SCOTUS says, Christian burial speech is the equivalent of an interrogation, purpose of speech was to get him to speak. Deliberately eliciting doesn't mean questioning, it fits, but the definition is broader than that. 2nd argument: voluntarily waived his rights by speaking. Acquiescence to police pressure is not sufficient for waiver. He had continually taken steps to retain his rights.

Ross v Moffitt

An indigent defendant is not entitled to representation at state expense for discretionary appeals. Due process requires that the state provide representation to all defendants during trial proceedings. This rule aims to promote fairness between the individual and the state in a situation in which the state seeks to alter the legal relationship between the parties. By contrast, the state is not required to provide appellate review. When the state chooses to afford the opportunity for appellate review, equal protection demands that it afford equal opportunity for review to all defendants, irrespective of the defendant's ability to pay the costs of appellate review. The state is not required to provide counsel at every step of the appellate process in order to provide meaningful access to the system. The state is simply prohibited from adopting arbitrary standards that exclude an individual from the opportunity to receive fair appellate review. When a defendant has been represented by counsel at both the trial level and upon the taking of an appeal of right, a sufficient record will have been established to enable review by a superior court. A defendant proceeding pro se before the superior court will be at a disadvantage relative to one who can afford representation, but that disadvantage is much less significant than it would be during a trial or during an initial appeal of right. In addition, the scope of review afforded by the state and federal supreme courts is discretionary. The supreme courts are not bound to review every case in which the appropriateness of a lower court judgment is called into question. The Supreme Court of North Carolina limits its review to issues of significant public interest and questions of conflict with prior supreme court decisions. The state supreme court has discretion to deny certiorari of any petition not presenting one of its criteria for review. This Court possesses the same discretionary authority and has consistently exercised a policy of denying appointed counsel for petitioners seeking certiorari. • NOTES: where there are multiple, discretionary appeals don't have to give you 6th amendment rights to council. Ensures trial was fundamentally fair for presumed innocence. After first appeal, NO LONGER presumed innocent.

When defendant is going to lie?

Attempt to dissuade, remove yourself from counsel, avoid questions on stand, tell judge

If the statement is a violation of the 5th amendment, it cannot be used for any purpose during trial.

Be careful! Is the subject in custody? Not in custody, not miranda. Can crossexamine on decision to remain silent.

Battle of antietam (3 arches bridge)

Burnside's bridge, 500 confederate soldiers able to hold off the union army for several hours here during the battle of Antietam.

=-- 1st confession admissible if public safety exception.

Cross examination of witness if they lie on the stand to prove unreliableness if mere miranda. Coerced confession is inadmissible under all circumstances (inherently unreliable and 5th amendment violation).

Farretta V California

Defendants have the constitutional right to represent themselves at trial. First, such a right is implied in the Sixth Amendment. The Sixth Amendment outlines what a constitutionally complete defense requires. The rights outlined in the amendment-the right to notice, confrontation and compulsory process-are personal rights given to the defendant to make his own defense. He can therefore represent himself if he decides it would be the best course of action. Furthermore, the plain language of the Sixth Amendment speaks to the "assistance" of counsel. If counsel is imposed on the defendant, the defendant is no longer in charge of his own defense but is subject to the will of the attorney. Second, history supports the position that states may not impose counsel on unwilling defendants. From the common law of the seventeenth century, to the American colonies, self-representation has been the norm. While the Founders saw the benefit of having counsel present, there is no indication that they intended that the right to counsel be a compulsory one. In addition, defendants have had the right to represent themselves in federal courts since the United States was founded and most states today allow defendants to do so as well. Finally, forcing counsel on an unwilling defendant could actually hurt his case. The defendant could become convinced that the law works against him. Also, there are times when a defendant could actually represent himself more effectively than an attorney could. Therefore, defendants may knowingly and intelligently waive their right to counsel. This is what Faretta did in this case. He made it clear to the judge that he wanted to represent himself. He was literate, competent, understanding, and had been warned of the dangers of representing himself. He was therefore denied his constitutional right to conduct his own defense when the state courts forbade him to represent himself. • NOTES: waiver doctrine applies: Right to representation is the right to assistance of council. Still the defendant's trial, defendant's right. Knowing, voluntary, and intelligent waiver. Doesn't count if illiterate, competency, etc. Right to self-represent is not absolute. Not limitless for self-representation can be lost involuntarily.

Incarcerated, can police deliberately illicit and incriminating statement?

Depends (pretrial or post first critical stage?)

Katz v U.S.

Even when there is no physical invasion, wire-tapping a public phone booth is a Fourth Amendment search and seizure. The Fourth Amendment protects a person against unlawful government intrusion; it is not intended to provide constitutional protection to a specific place. Katz was justified in assuming that his phone conversation would remain private, even though the phone booth is at all other times for public use. Therefore, Katz was protected under the Fourth Amendment when he entered the phone booth and shut the door, and the unwarranted recording of his phone conversation constitutes a search and seizure under the Fourth Amendment. NOTES: Public telephone booth. Gov attached wire tap to the outside of the phone booth. Motion to suppress under 4th amendment. Statements come in at trial. Katz appeals. Appeals lets the statements in. SC reverses. Gov says they didn't go into the booth so they didn't trespass. BUT court says it doesn't matter if you trespassed, you don't need physical intrusion for the 4th amendment. Gov argues the booth was glass, people could see him, so not protected BUT since he closed the door in the phonebooth he was trying to preserve himself from people listening. 4th amendment protects people not places. Extends 4th amendment to protect a reasonable expectation of privacy. Reasonable expectation of privacy—applies whether subjective or objective, but a phone booth counts as private. NOTES: replaces trespass with reasonable expectation of privacy

Missouri v Frye

In Strickland, the Court held that (1) a defense counsel's performance must be deficient and (2) the deficiency must have prejudiced the defendant in order to find that the defendant did not receive a fair trial as guaranteed by the Constitution. In addition to the Court's holding in Strickland, the Missouri Court of Appeals relied on Hill v. Lockhart, 474 U.S. 52 (1985), and Padilla v. Kentucky, 130 S.Ct. 1473 (2010), when it concluded that Frye's attorney was ineffective because he failed to communicate the prosecution's formal plea offers to Frye. In those decisions, the Court makes clear that defense counsel has a duty to communicate to a criminal defendant plea offers provided by the prosecution. Here, the State argues that a criminal defendant has no right to a plea offer. Weatherford v. Bursey, 429 U.S. 545 (1977). Nevertheless, plea offers and deals are integral to the criminal justice system. There are more plea offers accepted by criminal defendants than there are trials. To show that he has been prejudiced from ineffective assistance of counsel due to an un-communicated plea offer, a defendant must demonstrate a reasonable probability he would have accepted the prosecution's plea offer had it actually been communicated to him. Additionally, the defendant must show a reasonable probability that the plea offer would have been accepted by the trial court without the prosecution first revoking the offer. Under Hill¸ a defendant must demonstrate that but for defense counsel's errors, the defendant would not have pled guilty and instead would have gone to trial. The same logic may be applied to acceptance of plea offers. While the Missouri Court of Appeals correctly held that Frye's attorney was deficient by failing to communicate the plea offer to Frye it did not articulate the correct standard for prejudice. Although there is a strong likelihood that Frye would have accepted the plea offer provided by the prosecution, it is less certain that the trial court would have permitted the plea offer to become final. • NOTES: same Strickland test; Error: failure to convey plea offer. Depriving defendant constitutional rights to choose. 2nd part: whether or not(reasonable probability) it would have changed the outcome? ENDNOTE: state argues that a fair trial wipes the slate clean. SCOTUS rejects. Would have upended the 2 part test.

Powell v Alabama

In capital cases, where a defendant is unable to employ his own counsel and cannot adequately represent himself, a trial judge must appoint counsel, whether requested or not by the defendant. The Due Process Clause of the Fourteenth Amendment guarantees criminal defendants the right to notice and a hearing. In capital cases, the right to a hearing includes the right to counsel because a layman is generally not familiar with legal proceedings and is therefore not adequately prepared to assert his own defense. This right to counsel includes the ability to consult with one's attorney prior to trial in order to properly prepare a defense. In this case, the trial judge violated the Due Process Clause when he failed to give Powell and the other men the opportunity to employ their own counsel. Their youth, ignorance, illiteracy, and the severity of the charges make it clear that Powell and the others could not have effectively represented themselves at trial. Furthermore, by merely appointing "all members of the bar" for the "purpose of arraigning the defendants," the judge failed to assign proper counsel because no specific attorney was given clear responsibility of the case until the day of trial. There was therefore not enough time for Powell to prepare a proper defense in violation of the Due Process Clause of the Fourteenth Amendment. • NOTES: 6th amendment right to counsel. Rights to due process were violated. Denied council in a fair trial. Council wasn't informed on the facts of the case, not prepped for case. Illiterate, young men. AL said 6th amendment doesn't apply to the state. Didn't require council for indigent. BUT they DO have to because of 14th amendment. Has to appoint council if the person is illiterate. DOESN't say when council should be appointed. Right to council is FUNDAMENTAL. • ADDED: time before trial is applicable to the 6th amendment right to council • NOTES: entitled to council if jail time is possible. Either give them council, or waive jail time. Jail time doesn't have to immediately be imposed, to still be implicated. deferred incarceration is the same as incarceration. Appointment of council is the right to effective council. Lawyer is not expected to know everything about every single defense.

sell v US

NOTES: can we forcibly medicate someone to force them into standing for trial. Involuntary medication can't be used to control disruptive people in court not a mental health issue, not medically appropriate. Factors: significant gov interest (prosecuting criminal), necessary to further gov interest, unlikely to have side effects that undermine competency for trial (capable of being able to defend themselves), medically appropriate. Corollary: also have to be competent of being executed. Need to be able to understand what is happening to you.

PA v Muniz

NOTES: what pieces of evidence do the police have that they seek to admit when pulled over? Smell of alcohol, eyes bloodshot, words slurred—admissible? As long as probable cause/RAS to stop them, in plain view, all tangible evidence, covered by 4th amendment. Police gets guy out of car. What are we trying to get into evidence now? Dash cam recording and recording device on officer connected to the dash. Issue in muniz for 5th amendment? Sobriety test is physical evidence but not testimonial so not 5th amendment, 4th amendment. Physical evidence and video tape comes in. SO does audio come in? Officer was explaining the sobriety tests and asks if defendant understands. He says yes. IS that admissible? YES, speech quality is not testimonial. IS the yes itself testimonial? NO because the officer isn't looking for an incriminating statement at that point. Miranda doesn't apply/5th amendment. Defendant doesn't say yes, says I couldn't do that sober, implies drunk; could you repeat it I'm drunk—5th amendment doesn't apply because no one asked him, not responsive to the question, not coercing the statement, it is VOLUNTARY • Police station—biographical questions and what was the date of your 6th birthday? Routine/administrative questions, testimonial, but an exception. Routine booking exception. Required to ask in order to process you. So biographical questions allowed. 6th birthday question not allowed is testimonial, 5th amendment applies, no booking reason for question, clear attempt to determine sobriety/ incriminate him. • How do we know when something is testimonial? Suspect is required to assert an express assertion of fact. It is testimonial whenever the defendant faces the trilemma. Giving a truthful answer, lying, or remaining silence or admitting you don't know the answer. • 3 approved field sobriety point toe and count, heel to toe walk

6th amendment table

Pre 1st critical stage Post 1st critical stage Line-ups IN (not under interrogation) OUT Show-ups IN OUT (don't know but most likely yeah) Photo arrays IN IN --apply analysis for answer could go either way for ones not explicitly stated.

Georgia v McCollum

Primary restriction on batson prohibits state from making a strike on race and gender. Doesn't apply to defense. BUT NOW: defendants can't make those same strikes. Who can violate your constitutional rights? State or federal actors. Defendant is not either. Constitution says that you are entitled to a trial of your peers, doesn't mention preemptory challenges. Court gives you those. When a defendant uses a preemptory challenge, they are exercising an essentially governmental function, so they become gov actors, so they can't violate the constitutional rights of jurors, and therefore have to follow the same rules as the prosecution. Gov bestows gov authority and if they do so they have to act in accordance with the constitution.

Sequential admission case standard for confession admissibility?

Reaonably effectiveness of miranda

Olmstead

SCOTUS establishes trespass doctrine-->if the police don't intrude on your property, then they aren't violating your 4th amendment rights

Kuhlmann v Wilson

Statements made by a prisoner to a police informant who only passively listened and made no effort to elicit or induce those statements are admissible at trial. Henry held that statements made by a prisoner to a paid government informant should have been excluded under the rule set forth in Massiah v. United States, 377 U.S. 201 (1964). Nevertheless, Henrydid not address whether statements made to a police informant who listened but did not prompt, ask questions, or in any way attempt to elicit those statements were admissible. The purpose of Massiah was to prevent police from using covert interrogation techniques to circumvent the Sixth Amendment. There must be some deliberate action on the part of police for a violation of the Sixth Amendment to occur. Thus, there was no violation in this case, because police instructed the informant to listen to Wilson but avoid asking questions. • NOTES: takeaway: statements made to anyone spontaneously (not asking questions) the incriminating statements are admissible. Post-first critical stage, but makes no difference.

Gideon v Wainwright

The Fourteenth Amendment incorporates the Sixth Amendment right to counsel to the states. The Fourteenth Amendment incorporates those provisions of the Bill of Rights that are "fundamental and essential to a fair trial." The holding in Betts v. Brady, 316 U.S. 455 (1942), assumed that the state court's refusal to appoint counsel did not violate such fundamental principles of fairness and that there was no due process violation. On reconsideration, however, it is clear that Betts should now be overruled. Not only is it not good law, but even when it was decided it was not consistent with precedent. Powell v. Alabama, 287 U.S. 45 (1932), held that the right to counsel is fundamental. At the time, the Powell holding was limited to its facts. However, what it said about the fundamental nature of the right to counsel must now be embraced. Defendants have the constitutional right to a fair trial and this requires having an advocate present who knows the intricacies of the legal system. Accordingly, the decision in Betts is now overruled. • NOTES: 6th amendment right to council IS a fundamental right. Overturrns Betts. Lawyers in criminal cases are necessary, NOT a luxury

U.S. v White

The Fourth Amendment does not prohibit government agents from testifying to what they heard over a wire-tap worn by an informant. The Court has held that police can write down notes about a conversation they have with a defendant while undercover and testify to those transactions. For constitutional purposes, there is no distinction between immediately writing down these transactions and simultaneously recording or relaying the conversation to agents through electronic devices. While in Katz v. United States, 389 U.S. 347 (1967), the Court held that the use of a recording device on the outside of a phone booth amounts to an unconstitutional search because the user of the phone booth has a justifiable expectation that his conversation will remain private, the Katz decision does not alter this rule. Here, unlike in Katz, the defendants assume the risk when they confide in others about their illegal activities. Therefore, electronic surveillance that allows agents to listen in real time is admissible provided the agent is not otherwise violating the defendant's reasonable expectations of privacy. • NOTES: No warrant needed because none of it took place in a constitutionally protected area. Simultaneous recordings don't mean anything. Be careful who you trust! Expectation that a colleague won't reveal info to police is not protected by the constitution. • ADDED: misplaced trust doctrine—you give away your privacy in what you give out to the public in your voice. No one has a reasonable expectation of privacy that their convos are private. • NOTES: what role do electronics play in the analysis of the 4th amendment issues with police officers? A couple scenarios. Informant does drug deal, takes info back, carries to police, and turns it over. 2nd what happens if informant brings police in, hiding there listening itself. NO DIFFERENCE. Issue of fact, not law. What happens if the defendant is wearing a wire, broadcasting to officers v carrying out a tape recording. SAME THING. ALL MISPLACED TRUST.

Hoffa v U.S.

The Fourth Amendment does not protect a wrongdoer's misplaced belief that a person to whom he voluntarily confides his wrongdoing will not reveal it. In such case, the wrongdoer has no legitimate interest at stake that the Fourth Amendment protects. In the case at bar, Hoffa was not relying on the security of his hotel suite—something that the Fourth Amendment protects from unreasonable search—when he made the incriminating statements. Rather, he was relying on his misplaced belief that Partin would not divulge his incriminating statements to law enforcement. The risk of speaking to a government informant is "the kind of risk we necessarily assume whenever we speak." As a result, the lower courts properly admitted into evidence statements that the defendants made to Partin, as well as Partin's testimony. NOTES: claims that since the gov employee was present that it violated 6th amendment right to council. Court doesn't buy it because their convo wasn't actually about a defense that they were doing for the first trial. ADD: he invited him in the suit not violation the 4th. 6th amendment right claim attaches at the first critical stage. When you've been indicted. Not indicted, no 6th amendment right to council. 6th amendment has to attach. Gov acts too soon, the claim is dismissed. 5th amendment right to attorney when interrogation happens. 6th amendment DOES NOT attach until gets to a critical stage. NOTES: really pay attention to this case. 4th amendment, 5th amendment 2 6th amendment and 14th amendment question. 4th misplaced trust. 5th later. 6th 2 separate trials. Hoffa already on trial for racketeering. During that trial, tries to bribe jurors. Not convicted in first trial. Subsequently tried for bribing jury, new case. When they put a gov informant in hotel after 6th amendment rights have attached, did it violate right to counsel yes, but no remedy. Never convicted so can't suppress the evidence and no new trial because not convicted. 2nd case 6th amendment right not attached yet. When does 5th amendment right to council attach/6th amendment right attach? Hoffa's claim fails.

U.S. v Jones

The Fourth Amendment provides, in part, that the people are to be secure in their persons, houses, papers, and effects, against unreasonable searches and seizures. A vehicle is an "effect" for Fourth Amendment purposes. Thus, the government's installation of a GPS device on Jones's vehicle constitutes a "search." The issue is whether the placement of the GPS device on Jones's vehicle to monitor his movements was done in violation of the Fourth Amendment. Historically, the Court's Fourth Amendment jurisprudence was property-based and reflected the notion that a person had a right to be free from unreasonable searches and seizures in their persons, houses, papers, and effects." Later, the Court deviated from that exclusively property-based approach and began to hold that the Fourth Amendment "protects people, not places." Katz v. United States, 389 U.S. 347, 351 (1967). Here, the Government's warrantless placement of a GPS device on Jones's vehicle constitutes an unlawful search in violation of the Fourth Amendment. In making that decision, the Court applies not only the approach articulated in Katz, namely a person's reasonable expectation of privacy, but also incorporates the property-based trespass test articulated earlier.NOTES: Violation of reasonable expectation of privacy for length of time that they monitored (concurrence). Violates a trespass doctrine—majority. ADDS: violated trespass doctrine by touching the car. Reasonable expectation of privacy doesn't exist in open field. Careful of whether car enters a different jurisdiction then they wouldn't have the authority to look at it. NOTES: scalia says katz never overturned the trespass doctrine it layered it on top. Most cases never need to get to trespass. If physical intrusion on property, then violation regardless of katz.

Lewis v U.S.

The Sixth Amendment does not entitle a criminal defendant to a jury trial on petty offenses, even when conviction on multiple petty offenses could result in a sentence greater than six months in prison. The Sixth Amendment right to trial by jury attaches only to serious offenses. Petty crimes are not subject to the right of trial by jury. The determination of whether a crime constitutes a serious offense was formerly based on the nature of the offense and whether a jury trial was traditionally afforded under common law. In more recent times, the advent of statutory crimes without a common law foundation altered the analysis of the seriousness of an offense. We now look to the legislature's determination of appropriate penalties to assess whether an offense is a serious offense to merit a jury trial. In this case, the legislature established six months in prison as the maximum penalty for the offenses charged. The fact that multiple convictions might result in a longer sentence does not alter the legislature's characterization of the crime as a petty offense. Even if we held that characterization of the offense should be based on the maximum aggregate prison term, the prosecution could avoid jury trial by charging and trying the offenses separately. NOTES: Gave a fake name. No 4th amendment violation. Gov agent is allowed to give a fake name. Home was a place of business, so didn't have 4th amendment protections. Undercover investigations happen all the time so it's definitely allowed. No search happened. Lewis gave him the drugs. The only thing seized were the drugs that he was given. The officer was invited into the house. It's Lewis' own fault for believing. Not entrapment because he wasn't coerced to do anything, willingly gave the drugs. NEED: by inviting people into home, no subjective expectation of privacy. Only have it when you protect it, so not when you invite others in. Business purposes counts. Officer can lie about who they are. Notes: misplaced trust doctrine. Inviting someone in is giving up privacy for the purposes of the invitation. Mistakenly rely on individual with privacy is your own fault. Aside: you are giving up right of privacy for the purposes of the invitation.

JDB v North Carolina

The age of a child subjected to police questioning is relevant to whether the child is in custody under Miranda. Whether a suspect is in custody for Miranda purposes is an objective inquiry. The age of a child subjected to police questioning is relevant to whether that child is in custody, so long as the age was known to the officer or would be objectively apparent to a reasonable officer. There are many aspects of law in which children are treated differently from adults, and a custody analysis should be no different. Simply put, a reasonable child will feel more pressure than a reasonable adult to submit to police questioning. Failing to distinguish between them would deprive children of their constitutional rights. Moreover, this case exemplifies the absurdity of treating children with the same standard as adults. Without taking age into account, law enforcement and courts use a reasonable person of average years for custody analysis. J.D.B. was pulled out of his seventh-grade classroom to be questioned, so the analysis would have to take into account how a person of average years would react to being pulled out of a seventh-grade classroom. This analysis is impracticable. J.D.B.'s age should have been taken into account when determining whether or not he was in custody while the police were questioning him. Because this was not considered, the judgment of the Supreme Court of North Carolina is reversed, and the case is remanded to determine whether J.D.B. was in custody taking into account all relevant factors, including his age. NOTES: adds that age of child needs to be considered in determining whether they're in custody.

moran v burbine

The fact that the police did not tell Burbine about his attorney's call does not affect the validity of his waiver. To be valid, a defendant's waiver of his Miranda rights must be voluntarily, knowingly, and intelligently given. Burbine argues that his ability to knowingly waive his rights was impaired because the police failed to tell him that his attorney called. Things that a defendant is unaware of have no effect on his ability to knowingly give up his Constitutional rights. Burbine understood that he had the right to remain silent and request an attorney. His waiver was not coerced and he knew that his statements could be used against him. Therefore, his waiver was valid and the court of appeals' holding is erroneous. Burbine also argues that Miranda should be extended to require reversal of a conviction where police mislead an attorney or fail to tell a defendant when his attorney tries to contact him. We decline to extend Miranda in this way. Miranda warnings are designed to protect the suspect's Fifth Amendment rights during interrogation and a rule regarding how police interact with attorneys does not further that goal. While requiring police to tell a defendant when his attorney calls would minimally further the goals of Miranda, the complexity that adopting such a rule would add to the Miranda rules outweighs any benefit the rule would have. We also reject the defendant's argument that his Sixth Amendment right to an attorney requires the court to suppress his confession. The Sixth Amendment applies only after adversary judicial proceedings are initiated against a defendant; not, as Burbine argues, when an attorney-client relationship has formed. Lastly, we reject Burbine's argument that the alleged police misconduct was so offensive that it deprived him of due process. The police behavior in this case is not so extreme to justify federal interference in state criminal proceedings. NOTES: don't have to inform him that an attorney shows up on his behalf, don't have to tell the sister, don't have to say he's under interrogation right of the defendant. Waiver= knowing(know what the right is and that you're waiving it), intelligent (informed), voluntary (absence of coercion). NONE of these elements implicated by what's happening outside.

sullivan v louisiana

The majority observed the "prosecution bears the burden of proving all elements of the offense charged and must persuade the factfinder "�beyond a reasonable doubt' of the facts necessary to establish each of those elements."� "It is self-evident, we think, that the Fifth Amendment requirement of proof beyond a reasonable doubt and the Sixth Amendment requirement of a jury verdict are interrelated. It would not satisfy the Sixth Amendment to have a jury determine that the defendant is probably guilty, and then leave it up to the judge to determine (as Winship requires) whether he is guilty beyond a reasonable doubt. In other words, the jury verdict required by the Sixth Amendment is a jury verdict of guilty beyond a reasonable doubt."� The majority observed "[i]n [Chapman v. California], we rejected the view that all federal constitutional errors in the course of a criminal trial require reversal. We held that the Fifth Amendment violation of prosecutorial comment upon the defendant's failure to testify would not require reversal of the conviction if the State could show "�beyond a reasonable doubt that the error complained of did not contribute to the verdict obtained.' "� The Chapman standard recognizes that "�certain constitutional errors, no less than other errors, may have been harmless in terms of their effect on the factfinding process at trial.' Although most constitutional errors have been held amenable to harmless error analysis, some will always invalidate the conviction."� "Consistent with the jury trial guarantee, the question [Chapman] instructs the reviewing court to consider is not what effect the constitutional error might generally be expected to have upon a reasonable jury, but rather what effect it had upon the guilty verdict in the case at hand. Harmless error review looks, we have said, to the basis on which "�the jury actually rested its verdict.' The inquiry, in other words, is not whether, in a trial that occurred without the error, a guilty verdict would surely have been rendered, but whether the guilty verdict actually rendered in this trial was surely unattributable to the error. That must be so, because to hypothesize a guilty verdict that was never in fact rendered — no matter how inescapable the findings to support that verdict might be — would violate the jury-trial guarantee."� "[T]he essential connection to a "�beyond a reasonable doubt' factual finding cannot be made where the instructional error consists of a misdescription of the burden of proof, which vitiates all the jury's findings. A reviewing court can only engage in pure speculation — its view of what a reasonable jury would have done. And when it does that, "�the wrong entity judge[s] the defendant guilty.' "� Likewise, "[i[n [Arizona v. Fulminante], we distinguished between, on the one hand, "�structural defects in the constitution of the trial mechanism, which defy analysis by "�harmless error' standards,' and, on the other hand, trial errors which occur "�during the presentation of the case to the jury, and which may therefore be quantitatively assessed in the context of other evidence presented.' Denial of the right to a jury verdict of guilt beyond a reasonable doubt is certainly an error of the former sort, the jury guarantee being a "�basic protectio[n]' whose precise effects are unmeasurable, but without which a criminal trial cannot reliably serve its function. The right to trial by jury reflects, we have said, "�a profound judgment about the way in which law should be enforced and justice administered.' The deprivation of that right, with consequences that are necessarily unquantifiable and indeterminate, unquestionably qualifies as "�structural error.

Spano v NY

The petitioner was foreign-born, relatively young, and not very well educated and was subjected to skillful interrogation. He was questioned for 8 hours straight with only a slight reprieve at abnormal hours. His slowly mounting fatigue was used against him to force a confession. His reasonable requests to contact his attorney was denied. He was lied to by Bruno and yielded to his friend's entreaties to get him out of trouble. Petitioner's will was chipped away by official pressure, fatigue, and sympathy falsely aroused inconsistent with the 14th amendment. A grandy jury already had sufficient evidence to require the petitioner to come to trial for first-degree murder and they had an eyewitness. Only sought out the statement to convict him. With this intent, behavior has to be examined with close scrutiny according to the court. The circumstances here are enough to reverse the conviction. The tactics used by the police violated the petitioner's fundamental rights. NOTES: deliberately trying to fatigue the defendant to overcome his will to resist. Needs to be during normal business hours with breaks. Can't rotate out interrogators. Not allowed to use sympathy falsely aroused. General rule: interrogations during normal business hours, with breaks, police can't rotate through to be fresher than the defendant, can't exploit friendships to falsely arouse sympathy

US v Marion

The right to a speedy trial does not apply to pre-indictment delays. The Sixth Amendment guarantees a criminal defendant a speedy trial. By its plain language, the amendment only applies to a defendant once a criminal prosecution has been initiated. The amendment confers no rights on a person who has not been accused of a crime and imposes no duty on the government to conduct an investigation or bring a prosecution in a set timeframe. The language of the Amendment suggests that the framers did not intend to extend the protections to individuals not yet subject to criminal prosecution, and precedent does not support such an expansive construction. Before arrest, an individual does not bear the loss of freedom or other disadvantages associated with criminal prosecution. Pre-indictment delays may impair the defendant's ability to secure evidence and construct a defense, but that detriment does not justify redefining the right. Statutes of limitations bar prosecution after a set period of time and balance the government's interests against the potential prejudice to criminal defendants. Thus, it is not necessary to invoke the Sixth Amendment since statutes of limitations already protect the defendant's interests. While intentionally delaying indictment to disadvantage a defendant so that a fair trial is not possible could lead to dismissal on due process grounds, the circumstances requiring dismissal will require future adjudication. In this case, prosecution was not barred by the statute of limitations. Further, there has been no showing of prosecutorial intent to prejudice the defendants or any genuine prejudice, and therefore dismissal is not warranted on due process grounds. NOTES: until first critical stage hits, speedy trial doesn't apply. Statute of limitation varies by offense. Driven by legislation not constitution. Doesn't matter how long it takes gov to figure out who committed the crime. Speedy trial doesn't apply to appellate process.

texas v cobb

The right to counsel does not attach to the investigation of unrelated offenses arising from the same set of facts. McNeil v. Wisconsin, 501 U.S. 171 (1991), held that the right to counsel arises after a formal charge and does not attach to uncharged offenses. Some jurisdictions have interpreted this in conjunction with Brewer v. Williams, 430 U.S. 387 (1977), and Maine v. Moulton, 474 U.S. 159 (1985), to mean the right attaches to any charges bearing a factual relationship to the original offense. That interpretation does not control. In Brewer, a suspect was transported to another jurisdiction after arraignment. Along the way, police interrogated, and the suspect revealed the location of a second victim. The suspect had not waived his right to counsel, and the interrogation during transport violated his Sixth Amendment rights. The fact that the suspect had already been arraigned on one charge and the question of whether the right to counsel attaches to factually similar charges were not addressed in the opinion. Constitutional principles may not be inferred from an opinion if not actually considered. Moulton also offers no instruction. Moultonaddressed the right to counsel as exclusive to individual offenses. Cobb asserts that restricting the right to individual offenses will afford police free reign to interrogate suspects without representation, ignoring the Constitution's requirement that suspects be apprised of their rights before custodial interrogations. The state and the public have an interest in law enforcement interviewing suspects, even if suspected of other charges. Nothing in the Constitution impedes that interest. The right is not necessarily restricted only to charged offenses. Blockburger v. United States, 284 U.S. 299 (1932), defines the limits of the Double Jeopardy Clause, and the same test should be applied to define the scope of the right to counsel. Under Blockburger, if the same set of facts supports multiple charges without proof of any additional facts, the charges arise from the same offense. The dissent opines that the Blockburger test will be difficult to apply, but the test of close factual relationship appears more challenging. Investigating officers cannot have full knowledge of relevant facts during an investigation. Here, Cobb had been indicted for burglary when he confessed to the murder. He had not been charged with the murders, so no right to counsel arose. Under Texas law, burglary and murder require proof of different facts and are two distinct offenses under the Blockburger test. Cobb's invocation of his right to counsel for the burglary charge did not render interrogation about the murders a violation of the Sixth Amendment. Cobb's confession was admissible. NOTES: Burglary—post-critical stage. Defense: all crimes arising out of the factual circumstances were all in the same spot, even though other crimes (kidnapping, etc) weren't charged yet. SCOTUS: 6th amendment right to counsel doesn't apply. Doesn't apply because there's more facts that need to be proven. Can be on 5th and 6th amendment spectrums. 6th amendment right doesn't apply for all charges/proceedings.

Nix v Whiteside

There is no ineffective assistance of counsel when counsel informs a defendant that he must disclose perjury to the court because an attorney cannot allow a client to give false testimony. The Model Code of Professional responsibility, the Model Rules of Professional Conduct, and the American Bar Association require such disclosure. The state Code of Professional Responsibility allows withdrawal of representation when a client threatens to commit perjury. Furthermore, precedence supports this almost universal standard [Strickland v. Washington, 466 U.S. 668 (1984)]. Therefore, Robinson's actions were in line with accepted norms of professional conduct and did not deprive Whiteside of his Sixth Amendment right to counsel. Robinson continued to present the self-defense argument to the jury. His admonition merely prevented Whiteside from perjuring himself. Accordingly, there was no ineffective assistance of counsel. • NOTES: No right to testify falsely. What is attorney supposed to do when they know the defendant is supposed to lie? Convince client not to lie, ask judge to relieve them from council (can't say why because can't accuse for lying without him lying on stand first), (motion will be denied if the person can't say why because then new attorney won't know he's gonna lie), if person testifies attorney will ask questions to avoid/not facilitate the lie, if he lies, tell the court.

Andersen, Hubbel

Twists on compulsory self-incrimination. What does the 5th amendment privilege protect you against? Compelled self-incrimination. CO v Connelley mental health murder confession. Gov compulsion to testify against yourself. She wrote it without compulsion, not that it is used against her so can be used against her. Verbally saying or writing down something without gov intervention is admissible against you without violating 5th amendment right. Have to prove it was obtained legally and who wrote it.

simmons v US

Under the Due Process Clause, a lineup will only be deemed unconstitutional if the procedure used was so unfairly suggestive as to create a significant likelihood of an irreparable mistaken identification. Under Stovall v. Denno, 388 U.S. 293 (1967), any assertion that a particular lineup procedure violates due process must be evaluated in light of the totality of the circumstances. An unfair lineup may lead to witness misidentification that cannot be corrected later. The likelihood of misidentification is heightened if police show only a single photo, highlight a particular suspect, or indicate that outside evidence points to the suspect. This may taint the witness's memory and reduce the witness's reliability. Nevertheless, police have long used these lineup procedures to catch the guilty and clear the innocent. The defense may use cross-examination to demonstrate the shortcomings of the lineup procedure. There is no bright-line prohibition; rather, factual determinations must be made on a case-by-case basis. In this case, the FBI agents were investigating a serious felony and had not apprehended the suspects. The likelihood of misidentification was very small in light of the conditions of the robbery and the freshness of the witnesses' memories. Each of the witnesses identified Simmons without any prompting by the FBI agents, and none of the witnesses identified Andrews, even though Andrews was also in all the photos. In light of all of the surrounding circumstances, it is clear that Simmons was correctly identified even if the procedure used was not perfect. • NOTES: pre 1st critical stage photo arrays admissible without presence of council. Unless that photo array was unduly suggestive. Impermissibly gives rise to misidentification. What does unduly suggestive mean? Can have just one picture to show. Or an actual array. • FOOTNOTES IN post 1st critical stage

Garrity, Gardner, Kastigar

What do we do when the officer is the subject of the investigation? Does officer retain the right to council? Garrity—not given up from their job (5th amendment). Officers need to be read miranda. Officers can invoke miranda rights.

escobedo v illinois

When an investigation shifts from a general inquiry into an unsolved crime to a focus on a particular suspect, that suspect has been taken into police custody for questioning, the suspect has asked for and been denied his lawyer, and the police have not properly warned him of his right to remain silent, any confession made during the remainder of the interrogation is inadmissible. When the process moves from investigatory to accusatory, the accused must be given access to his lawyer. The fact that an interrogation under these facts may occur before the suspect is indicted makes no difference. In this case, the investigation began to focus specifically on Escobedo and Di Gerlando as the suspects. At this point, Escobedo was in custody and had asked to speak with his lawyer. The police's refusal to grant this request violates Escobedo's Sixth Amendment right to counsel and renders the subsequent incriminating statement inadmissible. Although it is said that the right to counsel attaching pretrial will be devastating to law enforcement because law enforcement obtains many confessions at that stage, this reasoning only fortifies the argument that the right to counsel should attach so early in the judicial process • NOTES: pre-1st-critical stage. Right to counsel also attaches when the process has gone beyond a general inquiry it shifted from investigatory to accusatory. SO 6th amendment right to counsel attaches

kansas v ventris

Where a defendant's Sixth Amendment right to counsel has been violated, the defendant's statement can still be admitted into evidence at trial for the purpose of impeachment. The Fifth Amendment guarantees that a person will not be forced to give evidence against himself and is violated whenever a coerced confession is introduced at trial. Similarly, the constitution is violated whenever a defendant is denied his right to counsel at trial in violation of the Sixth Amendment. However, the Sixth Amendment protects a defendant's right to counsel in certain pre-trial situations. Under this ancillary protection, when police violate a defendant's pre-trial right to counsel, statements and evidence are excluded only if the deterrent effect on police conduct outweighs the cost to society. In this case, Ventris' right to counsel was violated when he spoke with the informant. The need to prevent perjury and ensure the integrity of the trail are outweighed by his interests protected by excluding the evidence. Furthermore, excluding the evidence for the purpose of impeachment has little to no deterrent effect on the police. Police already want to abide by the law so that statements and evidence may be introduced as part of the prosecution's case-in-chief. NOTES: statements inadmissibly obtained can't be used in the case-in-chief, but can be used for impeachment if the defendant takes the stand and testifies falsely against the statement. What purpose does it come in? discredit the defendant, is he reliable, is he lying. Challenges the credibility of the defendant.

Transactional

complete grant of immunity from prosecution. Not given by mistake. Is a contract or agreement. Word agreement in advance because they want it to be a narrow agreement. Organized crime cases (mob, mafia).Person who committed execution is the one who gets transactional immunity which is the only way to get to the crime boss.

Andersen

defendant already has the document. State wants to compel the defendant to admit that this is their handwriting. While the stateements aren't compelled, if you are compelling them to acknowledge ownership, that's also testimonial; have to admit to assertion of fact, trilemma, 5th amendment applies. Handwriting expert, someone says they saw her write it, facebook/social media posts person is familiar with it and says that you wrote it, BUT can't force defendant to admit this is them.

6th amendment right to counsel attaches when shift from investigatory to accusatory

false

Confession inadmissible because not voluntarily given if showing mental illness

false

Need warrant for sense-enhancing device not readily available to public

false (4 part test!)

Sense-enhancing device makes misplaced trust not apply?

false (white)

5th amendment right to counsel attaches at 1st critical stage

false, attaches on custodial interrogation

Jail time potential outcome

get counsel

Alford plea

if defendant wants to plead guilty without admitting to having committed the crime

Robinson plea

no lo contede, no contest. Too drunk to remember what happened.

Consequences of an improper line up, show up, or photographic array

out of court ID inadmissible, in court may be admissible with substantial for basis of id

Purpose of miranda

overcome coercion

Immunity

privilege against being prosecuted to some extent. 2 types—use (derivative use immunity) and transactional immunity.

Gardner, Kastigar

public safety issue. Can't let officer continue to do job? Answers from officer necessary. How do we get answers while still protecting 5th amendment rights? If officer is the subject of a criminal investigation, yes they have rights. If they invoke, the investigation must stop, but the investigation may shift from criminal to administrative (as an officer). Can this person be employed anymore? In employment context the 5th amendment doesn't protect you. Administrative can insist that they answer questions truthfully. May change who asks the questions.

estelle v Williams

right to appear in street clothes for court

Error that prejudiced the defense

shows that council erred

Use

traditional; given by mistake or deliberately. To the extent that an officer ignore invoke of rights, if they use the term use immunity then it is inadmissible against defendant to prove their guilt. NOT immune from prosecution. Statement leads to evidence is also inadmissible. Substantial assistance—built into fed system, providing assistance will get people a lesser charge. Use it mostly for drugs or gangs. Can be used for anyone, not just an officer.

Gov official have license or consent to enter porch for routine matters

true

Ultimate question Kastigar

what penalties ensue to the person who refuses to answer? 5th amendment doesn't apply so can be fired for refusing to cooperate, answering falsely, or answering truthfully and admitting to the conduct. If officer invokes, pursue the administrative investigation, stuff can't be used in the criminal context. New attorney for both investigations. Not admissible at all (criminal investigation), unless person takes stand and lies.

Hubbel

whitewater, Clinton. Ponzi scheme. Hubbel gets prosecuted. He pleads guilty and is working with prosecution. Prosecution were going to revoke his status and punish him for not cooperating under the agreement. This is a business, so has records. But in any sizeable business, have thousands of documents. FBI doesn't want to read every doc, want hubbel to identify the incriminating docs, that incriminate hubbel. Wanted him to produce any doc in his possession, under his agreement, that is incriminating of himself. SCOUS says this is analogous with notebook. You can get a warrant and read the docs and whatever, but can't compel them to acknowledge that incriminating docs exist. Providing docs means acknowledging they exist, so is testimonial which is compulsory self-incrimination—not allowed.

doyle v ohio

• Rationale: A criminal defendant's due-process rights are violated if the trial court allows the prosecution to cross-examine the defendant about an exculpatory version of events that the defendant did not reveal to police after receiving Miranda warnings. The Miranda warnings are intended to protect the Fifth Amendment rights of the accused by informing the accused of his right to remain silent, his right to the assistance of counsel, and the fact that any statements he offers may be used against him. The Miranda warnings carry an implicit assurance that no penalty will attach to the exercise of the right to remain silent. Indeed, in this case, Doyle may have relied upon that implicit assurance as the basis for his decision to remain silent. Use of a defendant's silence for impeachment purposes, when the government has given the defendant reason to believe that his silence will not be used against him, violates the defendant's rights under the Due Process Clause of the Fourteenth Amendment. NOTES: miranda applies if in custody and under interrogation. Defendant invoked 5th amendment (I want to see lawyer/remain silent) This means police have to stop questioning until council or the defendant initiates the convo. Unless some emergency exception—public safety. If they invoke and remain silent and it applies, prohibited from holding that against the defendant. Different from defendant not in custody, not making statement, miranda doesn't apply. Can use silence against them. Additionally, defendant makes statement but claims they didn't, statement comes in for the purpose of proving false testimony. OR defendant remains silent and at trial and says that they said something so silence used against them. Can't come in as affirmative evidence. ENDNOTE

Berghuis v Thompkins

• Rationale: A defendant may implicitly waive his Miranda rights by failing to invoke his rights after fully understanding them and embarking on a course of conduct that indicates waiver. Thompkins argues that his statements are inadmissible because he invoked his right to remain silent by staying silent for a long period of time. This argument fails because a defendant must invoke his Miranda rights unambiguously. Here, Thompkins failed to unambiguously indicate that he wanted to remain silent. Thus, he failed to invoke his right to silence. Even where a defendant waives his right to silence, the prosecution must still establish that the defendant knowingly and voluntarily waived his right to remain silent before the statement can come in at trial. Waiver need not be express; an implicit waiver is enough. In North Carolina v. Butler, 441 U.S. 369 (1979), the Court established that waiver may implied by a defendant's silence where there is proof that the defendant understood his rights after being given the Miranda warnings and where his course of conduct indicates waiver. Here, Thompkins' understanding of his rights is unquestioned because he received a written form with the Miranda warnings and read part of the form aloud before the rest of the warnings were read aloud to him. Thompkins then embarked on a course of conduct indicating waiver by responding "yes" when asked if he prayed to God for forgiveness for shooting the victim. No evidence indicates that Thompkins' statement was coerced by police. Thus, Thompkins' waiver of his right to remain silent is established by the fact that he understood his rights and embarked on a course of conduct indicating waiver. Lastly, Thompkins argues that even if he waived his right to remain silent, the police were required to obtain a waiver prior to questioning. This is incorrect. Police may interrogate a suspect who has not yet chosen to waive or invoke his Miranda rights. Since Thompkins did not invoke his right to remain silent, but waived that right after fully understanding the Miranda warnings, the state court's decision to deny Thompkins' motion to suppress his statement was correct. NOTES: waiver has to be a free and deliberate choice. Made with full understanding of nature and rights of the situation. Freely, knowingly, voluntary—waiver.

North Carolina v Alford

• Rationale: A guilty plea can be accepted by the court even if the defendant testifies that he is innocent of the charges. A plea must be viewed in light of the evidence against the defendant to ensure that his guilty plea is intelligently made. If the state has a strong case against him, the defendant's guilty plea can be accepted by the court despite his testimony that he is innocent. Prior cases support this rule. Lynch v. Overholser, 369 U.S. 705 (1962), implied that a judge may accept a guilty plea even where there is evidence of a valid defense. Furthermore, since Hudson v. United States, 272 U.S. 451 (1926), courts have accepted nolo contendere pleas. In this case, the state had a strong case against Alford. He did not want to face the possibility of a death sentence if he went to trial and decided with the help of counsel that pleading guilty to second-degree murder was in his best interest. The strong evidence against Alford negates his claims of innocence and the judge therefore did not commit constitutional error in accepting his guilty plea. NOTES: alford pleas: not required by constitution. Any state can bar alford pleas and require defendant to admit conduct to give the plea . DE does allow alford pleas, even if allowed can reject the pleas. When defendant denies fact but want to enter guilty plea. Concern for ultimate penalty if he doesn't plea guilty. Want a lesser sentence. Pleas can be open or closed. Guarantees their certainty. Closed: sentence is part of the pleas, something that you agree to go in on. If judge goes back on it, you can rescind your plea. Pleading guilty to one charge so that all other charges will be dismissed. Open pleas: still get certainty of charges. Judge will only sentence you on one. Prosecution may make a sentence recommendation, but it is not obligative. Defendant may make recommendations to the sentence. Sentence isn't part of the bargain, can't rescind the plea. Sentencing ultimately left to the judges discretion. No contest pleas: defendant will not admit or deny but agree to go to trial due to the facts the prosecution has against them. Frequently offered in court. No contest plea not admissible in civil proceedings. Defendant pleads no contest because they're acknowledging that state has enough evidence that a jury could reasonably find them guilty but don't want to say their guilty. People unable to admit guilt, don't have conscious memory of the event.

illinois v allen

• Rationale: A highly disruptive defendant may be constitutionally removed from a criminal trial. Under the Sixth Amendment, a criminal defendant is entitled to confront any witness against him. Inherent in this guarantee is the right to be present during proceedings. Nevertheless, criminal trials must be conducted in a dignified manner. Trial judges have a degree of flexibility to deal with those who fail to meet the minimum standards of behavior. Although there is one method for maintaining order in every case, judges may constitutionally utilize (1) physical restraints, (2) contempt citations, or (3) removal of an unruly criminal defendant. Binding and gagging a defendant would eliminate disruptive behavior so that trial could continue in the defendant's presence. This may be constitutionally appropriate in extreme cases, but there are substantial drawbacks including risk of prejudice to the jury, inherent indignity of restraints, and reduction in the defendant's ability to assist his attorney. Next, criminal contempt sanctions may be used, but this may have little deterrent effect on defendants threatened with serious criminal penalties. Lastly, a judge may order a defendant's removal until he agrees to behave. In this case, Allen's conduct was disruptive and antagonistic enough to justify binding and gagging. Allen was given multiple warnings and allowed to return with a promise of good behavior. Allen gave up his Sixth Amendment right to be present. NOTES: Right to confront your accusers, but isn't an absolute right. You can give it up. When confronted with this wild defendant, 1) let them know that they could get kicked out so they can knowingly, intelligently, and purposely make their decision (waiver doctrine applies). 4 choices of judge when defendant continues: remove them, bind and gag, civil contempt, criminal contempt. 5th: using electronic stun devices Problems with every rule. Bind and gag: prejudice because can't participate in the case. Criminal contempt: to hold them in criminal contempt you've got to stop trial and have another trial, why would he cooperate with the 2nd trial? Civil contempt: penalty is hiding income, not paying child support, nothing in bank so no taxes, civil contempt so can incarcerate until you produce some amount of what you owe. It works. Stun device: will incapacitate person, but significant drawbacks. Standby counsel defendant very disruptive. Fires their attorney. They can represent themselves if they fit the criteria, but that doesn't preclude court from appointing someone to participate in the process, maintain knowledge level of case significantly, so if defendant is removed for something, they can represent you.

missouri v seibert

• Rationale: A second confession after a Miranda waiver is admissible only if there was a long enough break following the initial confession without a Miranda waiver to give a reasonable suspect the belief that he or she had a right not to speak to officers. The four-judge plurality agrees that both the subjective intent of the officers and the suspect's subjective understanding of his or her rights are irrelevant. In this case, Seibert's second confession was inadmissible despite her Miranda waiver because her first confession occurred without a Miranda waiver. Seibert's second confession was inadmissible despite her Miranda waiver because her first confession occurred without a Miranda waiver. Applying an objective standard, Seibert's first and second confessions were given during a single, continuous interrogation and thus her second confession was inadmissible despite her Miranda waiver NOTES: 1st confession, miranda, then 2nd confession. Standard: whether or not miranda rule was reasonably considered effective. If so, 2nd confession admissible. Rapid-fire confessions, no change in interrogator—weighs against admissibility. Can read, verbalize rights, understands enough to explain it—indications that rights warning was effective.

shepard v maxwell

• Rationale: A trial judge's failure to take steps to guard against prejudice caused by pretrial publicity violates the defendant's right to a fair trial by an impartial jury. A fair hearing by an unbiased jury is the hallmark of due process. Modern media coverage poses the risk of prejudice to jurors. Trial judges have a duty to take steps to protect the defendant from this prejudice. The press is entitled to report on ongoing trials. Nevertheless, when pretrial publicity threatens the defendant's fundamental rights, the trial court should issue a continuance or change the venue, sequester the jury, or order a new trial. In this case, the judge claimed that he had no authority to control the media. Nevertheless, the judge failed to take any of the available steps to guard against prejudice to the jury. The judge should have strictly limited the number and conduct of members of the media in the courtroom. Further, the judge should have barred witnesses from discussing their testimony with the media. Likewise, the judge should have prohibited police, attorneys, and other officials involved in the case from discussing the case and the evidence with the press. These measures were within the authority of the trial judge and would have greatly reduced the prejudice to the jury pool caused by the "carnival atmosphere at trial." By controlling the trial actors, the judge could have preserved Sheppard's right to a fair trial without curbing the rights of the press to report on the trial. NOTES: 2 extra credit question: what movie is made for this case? 2 movies 1 tv show. Judge is responsible to ensure that a defendant's constitutional rights are protected. Judge should have sequestered jurors, moved trial to different location, provided a continuance to let media coverage to die down and the election would have passed, control the release of info by issuing gag order, can't bar press from being there unless extreme circumstances, public right to know what's happening in courtrooms, but can limit how many media persons are there and not allowed to be at the bar

oregon v elstad

• Rationale: A violation of Miranda has no fruit except for the statement itself so a subsequent statement, made after the Miranda warnings have been read, is not tainted and is admissible at trial. Miranda violations differ from Fourth Amendment violations where the exclusionary rule applies because a Miranda warning is a procedural safeguard; it is not in itself a constitutional right. In addition, the goals of the exclusionary rule and the Fifth Amendment are not necessarily invoked by an initial violation of Miranda. The exclusionary rule seeks to deter illegal police conduct, and one of the purposes of the Fifth Amendment is to ensure trustworthy evidence. Neither concern is present in a case where a suspect is free to speak or remain silent after he has been read his Miranda warnings. Therefore, while a subsequent statement, made after Miranda has been given, will be inadmissible if it is coerced, there is no such presumption that arises due to the initial Miranda violation. However, if the unwarned statement was in fact coerced, the voluntariness of any subsequent statements would be suspect. However, here, there is no indication Elstad's initial statement was anything but voluntary. He then willingly waived his rights after being read his Miranda warnings and continued to talk to the police.

illinois v perkins

• Rationale: A voluntary statement made by an inmate to an undercover police officer does not require a Miranda warning to be admissible in court. Miranda v. Arizona, 384 U.S. 436 (1966), was concerned with threat to the Fifth Amendment's privilege against self-incrimination created by official custodial interrogations. In those situations, a suspect may feel coerced or compelled to answer questions or face harsher punishment and retaliatory behavior by officials. The element of coercion is not present when a suspect speaks freely to an undercover officer that the suspect believes to be a fellow inmate. The fact that the suspect is in custody does not mean that an undercover officer cannot investigate and gather information. Under Hoffa v. United States, 385 U.S. 293 (1966), such undercover investigations do not violate the Fifth Amendment. Miranda does not prohibit undercover activities that fall short of coercion, and voluntary confessions obtained through those activities are probative and admissible in court. Further, Miranda cannot be invoked to prevent the admission of statements made by a suspect bragging to those he believes to be his cellmates. In this case, Perkins believed the undercover officer was his cellmate and his equal. Consequently, from Perkins perspective, there was no reason to fear that the officer could compel Perkins to answer questions or in any way control Perkins' treatment. Thus, there was no violation of Perkins' Fifth Amendment rights. NOTES: miranda doesn't apply: not a coercive atmosphere, views him as an equal. Not in custody for the purposes of the 5th amendment. Purpose isn't served—coercive police. Pre first critical stage so 6th amendment doesn't apply.

US v salerno

• Rationale: An arrestee may be detained prior to trial if the government's regulatory interest in public safety is legitimate and compelling, provided there are procedural protections in place to safeguard the arrestee's liberty interests. A statute that permits an arrestee to be detained prior to trial is not inconsistent with the Due Process Clause of the Fifth Amendment or with the Excessive Bail Clause of the Eighth Amendment. With respect to due process, if the government has a legitimate and compelling interest in preventing crime, the state's regulatory interest outweighs individual liberty. For example, in Schall v. Martin, 467 U.S. 253 (1984), a statute was upheld that permitted the pre-trial detention of a juvenile who was arrested for any crime provided the state could show he was likely to commit some other crime. With respect to the Eighth Amendment's Excessive Bail Clause, the right to release on bail is not absolute. Moreover, the Eighth Amendment allows the government to pursue other compelling interests besides simply preventing flight of arrestees. This was made clear in Carlson v. Landon, 342 U.S. 524 (1952). In Carlson, the arrestees were all facing deportation and were detained pending trial because there was concern that their release would "endanger the welfare and safety of the United States." Therefore, although the Eighth Amendment prohibits excessive bail, it does not prohibit detention if Congress decides it is appropriate. In this case, although Salerno argues that the Bail Reform Act violates due process, this is not so. In enacting the statute, Congress intended to prevent crime by arrestees, which is a legitimate and compelling goal. The statute is a carefully crafted regulatory tool and does not impose punishment until the arrestee has a fair trial. Moreover, the statute has plenty of procedural safeguards in place to ensure that an arrestee is not needlessly deprived of his liberty. The prosecution has a high burden to satisfy, and the arrestee's rights are protected throughout the adversary hearing, as he may confront his accusers and have the assistance of counsel. Accordingly, the Bail Reform Act is not facially invalid under the Due Process Clause. Salerno also argues that the Bail Reform Act violates the Excessive Bail Clause of the Eighth Amendment, but this is also not the case. Salerno asserts that he has the right to bail unless it is likely he will fail to appear for trial. However, preventing Salerno from fleeing before trial is not the government's only concern here. Rather, the government's compelling interest in ensuring public safety is sufficient to warrant Salerno's continued detention, and the Eighth Amendment does not require his release on bail. NOTES: constitutional challenges can be made in 2 separate ways: statute is ambiguous on its face (facial challenge) no set of circumstances where it could be constitutional in any way. Unconstitutional as it is applied. NOT unconstitutional on its face not ambiguous, limited, not up to discretion of trial court. Most difficult challenge you could try to make. Unconstitutional as enforced by 8th amendment? Adds only 1 thing to stack v boyle: purpose of bail is to ensure community safety as well as securing defendant presence at trial. Proof positive hearing—prove that person is likely to be assigned death if found guilty so needs to be held without bail, also can forfeit their own right to bail.

neil v biggers

• Rationale: An identification made at an unfairly suggestive lineup is admissible if the identification is reliable. While an in-court identification is inadmissible if it is based upon a pretrial lineup so suggestive it was very likely to result in an irreparable mistaken identification, a pretrial identification is inadmissible if the procedure is so suggestive as to create a high likelihood of misidentification. The Court in Foster v. California, 394 U.S. 440 (1969), held identification evidence inadmissible because that likelihood of mistaken identification violates a criminal defendant's due process rights. Unfairly suggestive procedures heighten the risk of mistaken identifications. Nevertheless, under Stovall v. Denno, 388 U.S. 293 (1967), showup identifications are not necessarily unconstitutional. The question is whether an identification made after a suggestive lineup is nevertheless reliable. The totality of the circumstances, including the conditions of the identification and the witness's certainty, must be evaluated in each case. The decision of the district court in this case centered on the advantages of lineup identifications over showup identifications. Nevertheless, the conditions under which the victim observed the rapist, the victim's certainty, and the fact that the victim made no prior mistaken identifications all suggest that the identification evidence was reliable. In light of all the surrounding facts and circumstances, the likelihood of a mistaken identification was not significant in this case. NOTES: showup is inadmissible—not the big issue. Discusses opportunity to observe the person, dark night but moonlit, ample opportunity to view his face (15mins), she repeatedly rejected people thatweren't the rapist, she sees this person and days that's him without a doubt, extremely emphatic, no ambiguity of months of opportunities to misidentify. NEIL is about the circumstances under which the in court id is admissible even if out of court id is out. Victim can testify and say that he is the person who raped her and can say she is certain. Her credibility. Factors where in court is admissible—what was her opportunity to actually see the individual during the crime, independent basis for evaluation for admissibility. NOTE: court talks about how a significant amount of time had passed, normally overtime memories fade, multiple ops to misidentify, when she did identify, she was emphatic. Independent basis to identify—in court id in, out of court id is out.

stack v boyle

• Rationale: Bail is excessive and a violation of the Eighth Amendment when it is set at an amount greater than that necessary to ensure that the defendant will stand trial. In this case, the worst sentence the defendants could receive would be five years in jail and a $10,000 fine. The bail for offenses that carry a similar penalty is generally under $50,000. The government failed to show why such an excessive bail was necessary in this case to ensure that defendants appear for trial. The defendants can therefore move for a reduction of bail in the criminal proceeding NOTES: MAIN case. Bail wasn't fixed by proper methods everyone got the same bail, BUT should be INDIVIDUALLY TAILORED. Purpose of bail: secure the defendant's presence at trial. (this changes). What is bail designed to do? For the state? Insure that the defendant comes to court (generally same as court) Defendant? Not being detained, but also their defense at trial, harder to communicate with attorney, how they access evidence against them, privacy in viewing it, conversations with attorney may be monitored.

Douglas v California

• Rationale: During an appeal as a matter of right, an indigent defendant has the right to assistance of counsel. Denying an indigent defendant the right to counsel during an appeal as of right is a violation of the Fourteenth Amendment. A rich defendant can effectively appeal his case with the help of counsel. He can present his case to the court, and the court can make a decision based on the merits. A poor defendant, on the other hand, will be prevented from effectively exercising his right to appeal and instead only has the right to a "meaningless ritual." Therefore, Meyers and Douglas should have been appointed counsel for their appeal as of right. • NOTES: right to council on first appeal. No appointed attorneys for appeal because they had a rule that they would decide if the appeal had merit before they gave council. Denying defesne council on first appeal IS unconstitutional. Discriminated against poor people, so issue of equal protection of law.

Bordenkircher v Hayes

• Rationale: In North Carolina v. Pearce, 395 U.S. 711, 725 (1969), the Court held that the Due Process Clause of the Fourteenth Amendment to the U.S. Constitution "requires that vindictiveness against a defendant for having successfully attacked his first conviction must play no part in the sentence he receives after a new trial." However, there is no retaliation involved during the course of plea negotiations so long as the accused is free to accept or reject the prosecutor's offer. Although the prosecutor confronted Hayes with a possible life sentence if he chose to pursue his constitutional right to trial for the uttering charge, Hayes was free to make that decision. So long as the prosecutor has probable cause to believe that Hayes committed the uttering offense, the decision whether or not to prosecute, and what charge to file or bring before a grand jury, generally rests entirely in his discretion. NOTES: prosecutors: only a threat if they don't have the lawful authority to do it. Prosecutor has a legitimate interest in getting defendant to plead guilty/not guilty. If you do not have probable cause ofr a threat it is unconstitutional. Making a statement of fact that is within your prosecutorial authority is not a threat.

oregon v mathiason

• Rationale: Miranda warnings must be given only in situations where there has been such a restriction on a person's freedom as to render him "in custody." While any interview of a suspect by a police officer will involve some level of coercion, this does not mean the suspect is in custody. Where a suspect voluntarily answers an officer's questions, and he remains free to leave at any time, the suspect is not in custody. Therefore, the officer need not read him his Miranda warnings for the statements to be admissible at trial. In this case, Mathiason was never arrested, he came voluntarily, he only stayed for 30 minutes, and he was allowed to leave when he got up to do so. Any coercive tactics by the officer does not change the fact that Mathiason's freedom was not restricted in any significant way. Therefore, Mathiason was not in police custody and the officer was under no obligation to read him his Miranda warnings when he first arrived at the patrol office. NOTES: adds nothing to last case. EX that you can't be sucked in by location. Happens in p olice station, doesn't mean they're in custody.

US v Patane

• Rationale: Physical evidence found on the bases of a suspect's voluntary but unwarned statements is admissible at trial. Miranda warnings are required to protect a suspect's rights under the Self-Incrimination Clause, but admission of physical evidence found on the basis of a suspect's voluntary but unwarned statement does not violate the clause. The Self-Incrimination Clause guarantees that criminal defendants will not be forced to testify against themselves at trial. Nevertheless, courts have extended the protections of the Clause to allow suspects to refuse to answer questions that may later be used against them in criminal court. Because official custodial interrogations present such a grave risk to the privilege against self-incrimination, Miranda creates a presumption that statements made by a suspect who has not been advised of his rights are coerced for purposes of admissibility. This does not mean that police violate a suspect's constitutional rights simply by not giving the Miranda warnings. Rather, the suspect's rights are only violated if those unwarned statements are admitted at trial. Suppression of unwarned statements therefore cures Miranda violations. Thus, the fruit-of-the-poisonous tree doctrine is inapplicable unless physical evidence is found as a result of involuntary, coerced statements. Miranda's general presumption that unwarned statements are coerced serves only to protect a suspect's privilege against self-incrimination with respect to testimonial evidence, and therefore does not extend to physical evidence found as a result of voluntary, unwarned statements NOTES: defendant tells them where the gun is—"gun is under my bed"—inadmissible. Why is the statement a violation? Defendant is unmirandized. Was it coerced/voluntary? NO. SO MERE MIRANDA VIOLATION, NOTTTT 5th amendment. SO statement is admissible on cross-examination. NOT allowed in case-in-chief. Why doesn't fruit of the poisonous tree apply? Because scotus says it doesn't apply to the 5th amendment.

kirby v illinois

• Rationale: Police may conduct a lineup without an attorney present if the suspect has not yet been indicted or formally charged with a crime. Under United States v. Wade, 388 U.S. 218 (1967), and Gilbert v. California, 388 U.S. 263 (1967), a lineup conducted after arraignment is a "critical stage of the prosecution," which requires notification and/or the presence of the defendant's attorney under the Sixth Amendment. If this right is denied, identification testimony and in-court identifications must be excluded at trial. Nevertheless, the Sixth Amendment right to counsel does not attach until criminal prosecution has formally begun, signified by an arraignment or other judicial hearing. This marks the official beginning of the adversarial process and the activation of the Sixth Amendment. The Court will not extend the amendment's protections to events that occur before the initiation of formal proceedings. Abuses that occur during pre-arraignment identifications will be evaluated under the Due Process Clause of the Fifth and Fourteenth Amendments NOTES: showups staged. Defendant sitting with officer being booked/asked question and officer walks victim through the room, asks victim if they know them OR vice versa, victim sitting and walk defendant through the room. Pre-critical stage so no right to counsel under 6th. No right to counsel under 5th because he's not being interrogated.

Pena-Rodriguez v. Colorado

• Rationale: Rule 606(b) of Colorado's Rules of Evidence may not bar evidence of racial bias which is offered to prove a violation of the Sixth Amendment right to an impartial jury. Justice Anthony M. Kennedy delivered the opinion for the 5-3 majority. The Court held that, if a juror makes a clear statement that he or she convicted a criminal defendant relying on racial stereotypes or animus, the trial court is permitted to consider evidence of the juror's statements. The Court reiterated the sanctity of the jury to our criminal justice system and its reluctance to interfere with it. Rule 606(b), the no-impeachment rule, stems from the desire to prevent jurors from testifying about their deliberation after the verdict was entered, and it is a common law principle that is more lenient in some states than others. However, the Court noted that there was a possibility of an exception in the "gravest and most important cases." In certain rare cases, when racial animus is apparent in a juror, the no-impeachment rule shall be set aside in an effort to protect the Sixth Amendment. Not every "offhand" racial comment from a juror will warrant setting aside the no-impeachment rule. The exception demands that a juror exhibit overt racial animus that calls into question his or her ability to make a fair and impartial judgment about the defendant. Although the Court noted that there were processes currently in place to prevent racial bias in juries, such as the process of voir dire--the preliminary examination and elimination of jury members--these protections were often not sufficient, and therefore this exception was necessary. In this case, the Court held that the juror's statements were "egregious and unmistakable in their reliance on racial bias." The juror in question also encouraged other jurors to join in his racially-motivated conviction. NOTES: process expected: jurors take issue to counsel, counsel files a motion, wrapped into the appellate process. Jurors don't have a constitutional right to sit on the jury if they won't be impartial. Connection between race of defendant and juror doesn't exist—so unconstitutional if based on race on juror. Also for gender. Regardless of person's race or gender. State or defendant can assert Batson challenge. SCOTUS says court as a whole is harmed. Not only is the stricken juror harmed, denied of the ability to serve, state or defendant is harmed, community is harmed.

Howes v Fields

• Rationale: The Court stated that there was not yet any clearly established rule regarding what constituted Miranda custody. Mere imprisonment and private questioning about events in the outside world were not sufficient to create a custodial situation for Miranda purposes. Furthermore, the prisoner in this case was not in custody under Miranda because he was told at the outset of the interrogation that he could leave and go back to his cell whenever he wanted and because he was not physically restrained. NOTES: fact that person is in prison cell, doesn't mean they're in custody under miranda. Custody means not free to ignore police and go about their business.

Batson v Kentucky

• Rationale: The Equal Protection Clause is violated where the prosecution excludes African American veniremen from the jury based on nothing more than a presumed bias in favor of an African American defendant. Therefore, when the defendant makes a prima facie showing of discrimination during the peremptory challenge, the prosecution may be compelled to articulate a neutral explanation for why it challenges a particular veniremen. The right to peremptory challenges is not a constitutional one so there is no real difficulty in limiting its reach. Furthermore, the holding in Swain v. Alabama, 380 U.S. 202 (1965), has already established that peremptory challenges are subject to the principles of equal protection. Cases since Swain have established the standard for proving a prima facie case of purposeful discrimination. First, the defendant must show that he is a member of a specific racial group and that the prosecution has exercised its peremptory challenges to remove the veniremen who are of the same racial group. Then the defendant may rely on the fact that peremptory challenges are a practice that allows people to discriminate. Finally, the defendant must show that in light of all the surrounding circumstances, the prosecutor used the practice to exclude veniremen on account of their race. This inquiry need only involve evidence of the prosecutor's peremptory challenges at the defendant's own trial and need not establish a pattern of racial discrimination in prior cases. Once the defendant makes this prima facie showing, the burden shifts to the prosecution to justify its challenges with a neutral explanation. In this case, Batson made a timely objection to the prosecutor's use of peremptory challenges. NOTES: voir dire: 1st round of strikes is not discretionary (technically), removing them because they're not fit to serve on the jury. Removed for cause biased, can't speak English, medical issue. 2nd step is to select 12 people. 3rd is preemptory challenges. Purpose of preemptory challenges: trying to shape jury to their benefit. Can't strike juror for race IF they're the same race as the defendant and if they're a racial minority. Doesn't address gender or all races. ENDNOTE

J.E.B. v. Alabama ex rel. T.B.

• Rationale: The Equal Protection Clause of the Fourteenth Amendment prohibits peremptory challenges based on gender. There is a long history in this country of excluding women from juries. In this case, the state argues that its peremptory strikes were justified, because female jurors might be predisposed to sympathize with a single mother attempting to receive child support, while male jurors may be more likely to sympathize with a man who was the subject of a paternity challenge. This reliance on stereotypes is exactly the type of thing that equal protection seeks to avoid. When state actors engage in this type of discrimination, the effect is that the state seems to legitimize discrimination, and the efficacy of the court system comes into question. Discrimination in jury selection harms the litigants, the community, and the individuals who were not selected for jury duty based on impermissible factors. It leads to distrust of the legal system, and it must not be tolerated. NOTES: adds banning gender strikes on juries. What is connection between race and gender? Why add gender? Because it is an immutable characteristic, just like race women have been discriminated against purely for the stereotypical definition of gender. Strikes based upon characteristics such as race and gender, immutable characteristics, are unconstitutional when they are based upon individuals who have a history of invidious discrimination (denied constitutional rights like holding property or voting) based upon the various criteria. Can use a proxy to gender discriminate—but employment, religion, etc are not applicable under Batson (refers to both gender and race) if there's no pretext. Footnote 14

U.S. v Wade

• Rationale: The Sixth Amendment right to counsel is intended to afford the accused protection against state action during any critical stage of criminal proceedings, formal or otherwise, at which the right to a fair trial might be jeopardized by the lack of legal representation. A post-indictment witness identification is a critical stage of the proceedings. Witness identifications are notoriously unreliable, and conducting a lineup without the oversight of counsel allows the opportunity for suggestion to influence witness identification. A defendant's ability to identify infirmities in the identification process is severely limited, and the defendant labors under an inherent deficiency in credibility as compared to the police officers who would likely testify as to the soundness of the identification process. Here, the witnesses who identified Wade testified that they had seen him standing alone with the FBI agent before other lineup participants were brought in. Wade's conviction may have been decided well before he had the opportunity to present a defense at trial. The lineup clearly constituted a critical stage in the proceedings leading to his conviction, and the lineup should not have been conducted without notice to Wade's attorney or in his attorney's absence, unless Wade waived the right to have his counsel present. However, the absence of counsel at Wade's lineup does not necessarily require a new trial and the exclusion of the courtroom identification. Rather, the state should have the opportunity to prove that there was an independent source for the courtroom identification. In other words, the court must determine whether the courtroom identification arose exclusively from the impermissible lineup or whether it arose from circumstances sufficiently distinct from the lineup to remove it from exclusion as the fruit of illegal procedure. NOTES: pre trial line up post first critical stage, it's out. I out of court identification out? YES What ramifications does that have on the 2nd id in court?

rhode island v innis

• Rationale: Unless police officers reasonably should know that their comments will elicit an incriminating response from a suspect, comments made between police officers in the presence of a suspect do not constitute interrogation for the purpose of Miranda. Under Miranda, "interrogation" is not limited to situations where the police actually question a suspect. The Miranda opinion was concerned with the entire interaction between police officers and a suspect in custody. This includes direct questioning but also police techniques such as line-ups where a witness is coached by the police, and the various psychological ploys, such as blaming the victim. Miranda therefore applies when a suspect is subject to questioning or its functional equivalent. In this case, there is no indication that the two officers knew or should have known that Innis was particularly susceptible to an appeal to his conscience. Furthermore, there is no indication that Innis was disoriented or upset at the time of his arrest. Therefore, Innis was not interrogated for the purpose of Miranda because he was not subject to questioning or the functional equivalent. There is nothing in the record to suggest that the officers were trying to elicit a response from him or that they should have known their comments would elicit a response. • NOTES: no right to counsel because not being interrogated. No deliberate ploy to try to get the guy to confess. Officers were just talking amongst themselves. Different then Brewer (Christian burial). Brewer 6th amendment and this case is 5th.

Harris v NY

• Rationale: Unwarned statements may be used at trial for impeachment purposes. Miranda prohibits the prosecution from using unwarned statements to prove its case in chief. This serves to deter police from violating Miranda. Although dicta contained in the Miranda opinion suggests that unwarned statements may never be admitted, those pronouncements are not binding. In Walder v. United States, 347 U.S. 62 (1954), this Court held that evidence that had been barred from use in the government's case-in-chief by the exclusionary rule could be used to impeach the witness by an inconsistent statement. The fact that Walder was impeached on collateral matters and Harris was impeached on issues directly related to the crimes charged does not prevent application of the rule in Walder to this case. Reliable unwarned statements may be used at trial for purposes other than the prosecution's case in chief. Defendants are privileged against self-incrimination, but this does not mean that defendants may perjure themselves. Harris's testimony conflicted with earlier statements made to police. The prosecutor used those earlier conflicting statements impeach Harris' testimony, and those statements would certainly have been admissible if made to any other witness. Evidence of those statements was useful to jury members tasked with evaluating Harris' credibility. NOTES: statements made without miranda can't be used. Trustworthiness of evidence satisfies legal standards—not coerced in anyway. BUT otherwise voluntary—pure miranda violation so comes in on cross examination. Defendant has to take the stand and contradict the statement for it to come in. As long as officer is not coercive and the statement is voluntary—confession is admissible. FOOTNOTES.


Conjuntos de estudio relacionados

Chapter 14 Lesson 3 Southern Cotton Kingdom

View Set

Acc201 Chapter 6 Interactive Quiz

View Set

Ankle and Foot: Part 4 (Insufficient Dorsiflexion)

View Set

The Endocrine Pancreas and Glucose Regulation

View Set

Chapter 43: Assessment of Digestive and GI function Prep-U

View Set

Peds - Chapter 31: Health Supervision

View Set